why is the limit of this expression equal to 1? The 2019 Stack Overflow Developer Survey Results Are In Announcing the arrival of Valued Associate #679: Cesar Manara Planned maintenance scheduled April 17/18, 2019 at 00:00UTC (8:00pm US/Eastern)Finding the limit of the following expressionReforming series expression for limit of e$lim_x rightarrow inftyleft(fracpi2-tan^-1xright)^Largefrac1x$ Why aren't these two limits equal when they should be?What is the value of this limit?limit of an expressionUsing a definite integral find the value of $lim_nrightarrow infty (frac1n+frac1n+1+…+frac12n)$Why is the following limit operation valid?Is this expression on limit valid and/or meaningful?Why does this limit equal 0?A Problem on the Limit of an Integral

What was the last x86 CPU that did not have the x87 floating-point unit built in?

How do spell lists change if the party levels up without taking a long rest?

Does Parliament need to approve the new Brexit delay to 31 October 2019?

Why not take a picture of a closer black hole?

Why did Peik Lin say, "I'm not an animal"?

Can I visit the Trinity College (Cambridge) library and see some of their rare books

Huge performance difference of the command find with and without using %M option to show permissions

How can a C program poll for user input while simultaneously performing other actions in a Linux environment?

Visa regaring travelling European country

The following signatures were invalid: EXPKEYSIG 1397BC53640DB551

How to type a long/em dash `—`

How did passengers keep warm on sail ships?

Is 'stolen' appropriate word?

Deal with toxic manager when you can't quit

Is it ethical to upload a automatically generated paper to a non peer-reviewed site as part of a larger research?

Did the new image of black hole confirm the general theory of relativity?

What can I do if neighbor is blocking my solar panels intentionally?

Example of compact Riemannian manifold with only one geodesic.

One-dimensional Japanese puzzle

Accepted by European university, rejected by all American ones I applied to? Possible reasons?

Do I have Disadvantage attacking with an off-hand weapon?

Working through the single responsibility principle (SRP) in Python when calls are expensive

Did the UK government pay "millions and millions of dollars" to try to snag Julian Assange?

How did the audience guess the pentatonic scale in Bobby McFerrin's presentation?



why is the limit of this expression equal to 1?



The 2019 Stack Overflow Developer Survey Results Are In
Announcing the arrival of Valued Associate #679: Cesar Manara
Planned maintenance scheduled April 17/18, 2019 at 00:00UTC (8:00pm US/Eastern)Finding the limit of the following expressionReforming series expression for limit of e$lim_x rightarrow inftyleft(fracpi2-tan^-1xright)^Largefrac1x$ Why aren't these two limits equal when they should be?What is the value of this limit?limit of an expressionUsing a definite integral find the value of $lim_nrightarrow infty (frac1n+frac1n+1+…+frac12n)$Why is the following limit operation valid?Is this expression on limit valid and/or meaningful?Why does this limit equal 0?A Problem on the Limit of an Integral










1












$begingroup$


I found something which I find confusing.



$$
lim_nrightarrow infty fracn!n^k(n-k)! =1
$$



It was something I encountered while learning probability on this webpage.










share|cite|improve this question











$endgroup$
















    1












    $begingroup$


    I found something which I find confusing.



    $$
    lim_nrightarrow infty fracn!n^k(n-k)! =1
    $$



    It was something I encountered while learning probability on this webpage.










    share|cite|improve this question











    $endgroup$














      1












      1








      1


      2



      $begingroup$


      I found something which I find confusing.



      $$
      lim_nrightarrow infty fracn!n^k(n-k)! =1
      $$



      It was something I encountered while learning probability on this webpage.










      share|cite|improve this question











      $endgroup$




      I found something which I find confusing.



      $$
      lim_nrightarrow infty fracn!n^k(n-k)! =1
      $$



      It was something I encountered while learning probability on this webpage.







      limits






      share|cite|improve this question















      share|cite|improve this question













      share|cite|improve this question




      share|cite|improve this question








      edited 2 hours ago







      billyandr

















      asked 2 hours ago









      billyandrbillyandr

      155




      155




















          2 Answers
          2






          active

          oldest

          votes


















          5












          $begingroup$

          It is rather obvious if you cancel the factorials:



          $$fracn!n^k(n-k)! =fracoverbracen(n-1)cdots (n-k+1)^k; factorsn^k= 1cdot left(1-frac1nright)cdots left(1-frack-1nright)stackreln to inftylongrightarrow 1$$






          share|cite|improve this answer









          $endgroup$












          • $begingroup$
            Thank you so much. I didn't know it was right there under my eyes.
            $endgroup$
            – billyandr
            1 hour ago










          • $begingroup$
            You are welcome. This "not seeing the obvious" just happens once in a while, I think, to all who do maths. So, it is good to have a math platform like this one. :-)
            $endgroup$
            – trancelocation
            1 hour ago



















          2












          $begingroup$

          $$a_n=fracn!n^k(n-k)! implies log(a_n)=log(n!)-k log(n)-log((n-k)!)$$



          Use Stirling approximation and continue with Taylor series to get
          $$log(a_n)=frack(1-k)2 n+Oleft(frac1n^2right)$$ Continue with Taylor
          $$a_n=e^log(a_n)=1+frack(1-k)2 n+Oleft(frac1n^2right)$$






          share|cite|improve this answer









          $endgroup$












          • $begingroup$
            This has already a slight touch of overkill, hasn't it? :-)
            $endgroup$
            – trancelocation
            1 hour ago










          • $begingroup$
            @trancelocation. You are totally right for the limit. One of my manias is to always look at the approach to the limit. Have a look at matheducators.stackexchange.com/questions/8339/… . Cheers :-)
            $endgroup$
            – Claude Leibovici
            1 hour ago












          Your Answer








          StackExchange.ready(function()
          var channelOptions =
          tags: "".split(" "),
          id: "69"
          ;
          initTagRenderer("".split(" "), "".split(" "), channelOptions);

          StackExchange.using("externalEditor", function()
          // Have to fire editor after snippets, if snippets enabled
          if (StackExchange.settings.snippets.snippetsEnabled)
          StackExchange.using("snippets", function()
          createEditor();
          );

          else
          createEditor();

          );

          function createEditor()
          StackExchange.prepareEditor(
          heartbeatType: 'answer',
          autoActivateHeartbeat: false,
          convertImagesToLinks: true,
          noModals: true,
          showLowRepImageUploadWarning: true,
          reputationToPostImages: 10,
          bindNavPrevention: true,
          postfix: "",
          imageUploader:
          brandingHtml: "Powered by u003ca class="icon-imgur-white" href="https://imgur.com/"u003eu003c/au003e",
          contentPolicyHtml: "User contributions licensed under u003ca href="https://creativecommons.org/licenses/by-sa/3.0/"u003ecc by-sa 3.0 with attribution requiredu003c/au003e u003ca href="https://stackoverflow.com/legal/content-policy"u003e(content policy)u003c/au003e",
          allowUrls: true
          ,
          noCode: true, onDemand: true,
          discardSelector: ".discard-answer"
          ,immediatelyShowMarkdownHelp:true
          );



          );













          draft saved

          draft discarded


















          StackExchange.ready(
          function ()
          StackExchange.openid.initPostLogin('.new-post-login', 'https%3a%2f%2fmath.stackexchange.com%2fquestions%2f3185830%2fwhy-is-the-limit-of-this-expression-equal-to-1%23new-answer', 'question_page');

          );

          Post as a guest















          Required, but never shown

























          2 Answers
          2






          active

          oldest

          votes








          2 Answers
          2






          active

          oldest

          votes









          active

          oldest

          votes






          active

          oldest

          votes









          5












          $begingroup$

          It is rather obvious if you cancel the factorials:



          $$fracn!n^k(n-k)! =fracoverbracen(n-1)cdots (n-k+1)^k; factorsn^k= 1cdot left(1-frac1nright)cdots left(1-frack-1nright)stackreln to inftylongrightarrow 1$$






          share|cite|improve this answer









          $endgroup$












          • $begingroup$
            Thank you so much. I didn't know it was right there under my eyes.
            $endgroup$
            – billyandr
            1 hour ago










          • $begingroup$
            You are welcome. This "not seeing the obvious" just happens once in a while, I think, to all who do maths. So, it is good to have a math platform like this one. :-)
            $endgroup$
            – trancelocation
            1 hour ago
















          5












          $begingroup$

          It is rather obvious if you cancel the factorials:



          $$fracn!n^k(n-k)! =fracoverbracen(n-1)cdots (n-k+1)^k; factorsn^k= 1cdot left(1-frac1nright)cdots left(1-frack-1nright)stackreln to inftylongrightarrow 1$$






          share|cite|improve this answer









          $endgroup$












          • $begingroup$
            Thank you so much. I didn't know it was right there under my eyes.
            $endgroup$
            – billyandr
            1 hour ago










          • $begingroup$
            You are welcome. This "not seeing the obvious" just happens once in a while, I think, to all who do maths. So, it is good to have a math platform like this one. :-)
            $endgroup$
            – trancelocation
            1 hour ago














          5












          5








          5





          $begingroup$

          It is rather obvious if you cancel the factorials:



          $$fracn!n^k(n-k)! =fracoverbracen(n-1)cdots (n-k+1)^k; factorsn^k= 1cdot left(1-frac1nright)cdots left(1-frack-1nright)stackreln to inftylongrightarrow 1$$






          share|cite|improve this answer









          $endgroup$



          It is rather obvious if you cancel the factorials:



          $$fracn!n^k(n-k)! =fracoverbracen(n-1)cdots (n-k+1)^k; factorsn^k= 1cdot left(1-frac1nright)cdots left(1-frack-1nright)stackreln to inftylongrightarrow 1$$







          share|cite|improve this answer












          share|cite|improve this answer



          share|cite|improve this answer










          answered 1 hour ago









          trancelocationtrancelocation

          14.1k1829




          14.1k1829











          • $begingroup$
            Thank you so much. I didn't know it was right there under my eyes.
            $endgroup$
            – billyandr
            1 hour ago










          • $begingroup$
            You are welcome. This "not seeing the obvious" just happens once in a while, I think, to all who do maths. So, it is good to have a math platform like this one. :-)
            $endgroup$
            – trancelocation
            1 hour ago

















          • $begingroup$
            Thank you so much. I didn't know it was right there under my eyes.
            $endgroup$
            – billyandr
            1 hour ago










          • $begingroup$
            You are welcome. This "not seeing the obvious" just happens once in a while, I think, to all who do maths. So, it is good to have a math platform like this one. :-)
            $endgroup$
            – trancelocation
            1 hour ago
















          $begingroup$
          Thank you so much. I didn't know it was right there under my eyes.
          $endgroup$
          – billyandr
          1 hour ago




          $begingroup$
          Thank you so much. I didn't know it was right there under my eyes.
          $endgroup$
          – billyandr
          1 hour ago












          $begingroup$
          You are welcome. This "not seeing the obvious" just happens once in a while, I think, to all who do maths. So, it is good to have a math platform like this one. :-)
          $endgroup$
          – trancelocation
          1 hour ago





          $begingroup$
          You are welcome. This "not seeing the obvious" just happens once in a while, I think, to all who do maths. So, it is good to have a math platform like this one. :-)
          $endgroup$
          – trancelocation
          1 hour ago












          2












          $begingroup$

          $$a_n=fracn!n^k(n-k)! implies log(a_n)=log(n!)-k log(n)-log((n-k)!)$$



          Use Stirling approximation and continue with Taylor series to get
          $$log(a_n)=frack(1-k)2 n+Oleft(frac1n^2right)$$ Continue with Taylor
          $$a_n=e^log(a_n)=1+frack(1-k)2 n+Oleft(frac1n^2right)$$






          share|cite|improve this answer









          $endgroup$












          • $begingroup$
            This has already a slight touch of overkill, hasn't it? :-)
            $endgroup$
            – trancelocation
            1 hour ago










          • $begingroup$
            @trancelocation. You are totally right for the limit. One of my manias is to always look at the approach to the limit. Have a look at matheducators.stackexchange.com/questions/8339/… . Cheers :-)
            $endgroup$
            – Claude Leibovici
            1 hour ago
















          2












          $begingroup$

          $$a_n=fracn!n^k(n-k)! implies log(a_n)=log(n!)-k log(n)-log((n-k)!)$$



          Use Stirling approximation and continue with Taylor series to get
          $$log(a_n)=frack(1-k)2 n+Oleft(frac1n^2right)$$ Continue with Taylor
          $$a_n=e^log(a_n)=1+frack(1-k)2 n+Oleft(frac1n^2right)$$






          share|cite|improve this answer









          $endgroup$












          • $begingroup$
            This has already a slight touch of overkill, hasn't it? :-)
            $endgroup$
            – trancelocation
            1 hour ago










          • $begingroup$
            @trancelocation. You are totally right for the limit. One of my manias is to always look at the approach to the limit. Have a look at matheducators.stackexchange.com/questions/8339/… . Cheers :-)
            $endgroup$
            – Claude Leibovici
            1 hour ago














          2












          2








          2





          $begingroup$

          $$a_n=fracn!n^k(n-k)! implies log(a_n)=log(n!)-k log(n)-log((n-k)!)$$



          Use Stirling approximation and continue with Taylor series to get
          $$log(a_n)=frack(1-k)2 n+Oleft(frac1n^2right)$$ Continue with Taylor
          $$a_n=e^log(a_n)=1+frack(1-k)2 n+Oleft(frac1n^2right)$$






          share|cite|improve this answer









          $endgroup$



          $$a_n=fracn!n^k(n-k)! implies log(a_n)=log(n!)-k log(n)-log((n-k)!)$$



          Use Stirling approximation and continue with Taylor series to get
          $$log(a_n)=frack(1-k)2 n+Oleft(frac1n^2right)$$ Continue with Taylor
          $$a_n=e^log(a_n)=1+frack(1-k)2 n+Oleft(frac1n^2right)$$







          share|cite|improve this answer












          share|cite|improve this answer



          share|cite|improve this answer










          answered 1 hour ago









          Claude LeiboviciClaude Leibovici

          125k1158135




          125k1158135











          • $begingroup$
            This has already a slight touch of overkill, hasn't it? :-)
            $endgroup$
            – trancelocation
            1 hour ago










          • $begingroup$
            @trancelocation. You are totally right for the limit. One of my manias is to always look at the approach to the limit. Have a look at matheducators.stackexchange.com/questions/8339/… . Cheers :-)
            $endgroup$
            – Claude Leibovici
            1 hour ago

















          • $begingroup$
            This has already a slight touch of overkill, hasn't it? :-)
            $endgroup$
            – trancelocation
            1 hour ago










          • $begingroup$
            @trancelocation. You are totally right for the limit. One of my manias is to always look at the approach to the limit. Have a look at matheducators.stackexchange.com/questions/8339/… . Cheers :-)
            $endgroup$
            – Claude Leibovici
            1 hour ago
















          $begingroup$
          This has already a slight touch of overkill, hasn't it? :-)
          $endgroup$
          – trancelocation
          1 hour ago




          $begingroup$
          This has already a slight touch of overkill, hasn't it? :-)
          $endgroup$
          – trancelocation
          1 hour ago












          $begingroup$
          @trancelocation. You are totally right for the limit. One of my manias is to always look at the approach to the limit. Have a look at matheducators.stackexchange.com/questions/8339/… . Cheers :-)
          $endgroup$
          – Claude Leibovici
          1 hour ago





          $begingroup$
          @trancelocation. You are totally right for the limit. One of my manias is to always look at the approach to the limit. Have a look at matheducators.stackexchange.com/questions/8339/… . Cheers :-)
          $endgroup$
          – Claude Leibovici
          1 hour ago


















          draft saved

          draft discarded
















































          Thanks for contributing an answer to Mathematics Stack Exchange!


          • Please be sure to answer the question. Provide details and share your research!

          But avoid


          • Asking for help, clarification, or responding to other answers.

          • Making statements based on opinion; back them up with references or personal experience.

          Use MathJax to format equations. MathJax reference.


          To learn more, see our tips on writing great answers.




          draft saved


          draft discarded














          StackExchange.ready(
          function ()
          StackExchange.openid.initPostLogin('.new-post-login', 'https%3a%2f%2fmath.stackexchange.com%2fquestions%2f3185830%2fwhy-is-the-limit-of-this-expression-equal-to-1%23new-answer', 'question_page');

          );

          Post as a guest















          Required, but never shown





















































          Required, but never shown














          Required, but never shown












          Required, but never shown







          Required, but never shown

































          Required, but never shown














          Required, but never shown












          Required, but never shown







          Required, but never shown







          Popular posts from this blog

          Wolfenstein 3D Contents Availability Essential improvements Game data Video settings Input settings Audio settings Network VR support Issues fixed Other information System requirements NotesReferences    3D Realms Wolfenstein 3D pageGOG.com Community DiscussionsGOG.com Support PageSteam Community DiscussionsWolfenstein WikiOfficial websiteAmazon.comBethesda.netGamersGateGOG.comGreen Man GamingHumble StoreSteamweb browser versionWolfenstein 3D: Super UpgradesherehereUltraWolfhereWolfMenuECWolf Wolf4SDL WolfGL WinWolf3d NewWolf BetterWolf Sprite Fix and Rotation Project    Wolfenstein 3D VRSplitWolfWolfenstein 3D VRWolfenstein 3D VRWolfenstein 3D VR4DOS command shellFreeDOS's MORE.COMMacBin themthis shim fileWine regeditRELEASE: QUAKE II + III, WOLFENSTEIN 3D, RETURN TO CASTLE WOLFENSTEIN - GOG.com NewsMac Family - Wolfenstein Wiki - WikiaNerdly Pleasures: How many FPS? - DOS Games and Framerates

          Король Коль Исторические данные | Стихотворение | Примечания | Навигацияверсии1 правкаверсии1 правкаA New interpretation of the 'Artognou' stone, TintagelTintagel IslandАрхивировано

          Isurus Índice Especies | Notas | Véxase tamén | Menú de navegación"A compendium of fossil marine animal genera (Chondrichthyes entry)"o orixinal"A review of the Tertiary fossil Cetacea (Mammalia) localities in wales port taf Museum Victoria"o orixinalThe Vertebrate Fauna of the Selma Formation of Alabama. Part VII. Part VIII. The Mosasaurs The Fishes50419737IDsh85068767Isurus2548834613242066569678159923NHMSYS00210535017845105743